Difference between revisions of "Antiderivative of arctan"

From specialfunctionswiki
Jump to: navigation, search
Line 1: Line 1:
 
<div class="toccolours mw-collapsible mw-collapsed">
 
<div class="toccolours mw-collapsible mw-collapsed">
 
<strong>[[Antiderivative of arctan|Theorem]]:</strong> The following formula holds:
 
<strong>[[Antiderivative of arctan|Theorem]]:</strong> The following formula holds:
$$\displaystyle\int \mathrm{arctan}(z) \mathrm{d}z = z\mathrm{arctan}(z) - \dfrac{1}{2}\log(1+z^2)+C,$$
+
$$\displaystyle\int \mathrm{arctan}(z) \mathrm{d}z = z\mathrm{arctan}(z) - \dfrac{1}{2}\log \left(1+z^2 \right)+C,$$
 
where $\mathrm{arctan}$ denotes the [[arctan|inverse tangent]] and $\log$ denotes the [[logarithm]].
 
where $\mathrm{arctan}$ denotes the [[arctan|inverse tangent]] and $\log$ denotes the [[logarithm]].
 
<div class="mw-collapsible-content">
 
<div class="mw-collapsible-content">

Revision as of 04:16, 16 May 2016

Theorem: The following formula holds: $$\displaystyle\int \mathrm{arctan}(z) \mathrm{d}z = z\mathrm{arctan}(z) - \dfrac{1}{2}\log \left(1+z^2 \right)+C,$$ where $\mathrm{arctan}$ denotes the inverse tangent and $\log$ denotes the logarithm.

Proof: